Which of the following options have the same value as 40% percent of 84?
Choose 2 answers

Answers

Answer 1

Answer:

The answer in decimal form is 33.6.

The answer in fraction form is [tex]33\frac{3}{5}[/tex].

Step-by-step explanation:

To find 40% of 84, multiply 84 and 0.4.

84 × 0.4 = 33.6

The answer in decimal form is 33.6.

The answer in fraction form is [tex]33\frac{3}{5}[/tex].

Hope this helps!


Related Questions

Mercedes receives a $25 gift card for downloading music and wants to determine how many songs she can purchase. each downloaded song costs $1.29. write and solve an inequality to determine m, the number of songs mercedes can purchase. what is the approximate solution to the inequality? round to the nearest whole number.

Answers

The solution to the inequality: [tex]1.29m\leq 25[/tex]  is m=19.

What is inequality?

Mathematical expressions with inequalities are those in which the two sides are not equal. Contrary to equations, we compare two values in inequality. Less than (or less than or equal to), greater than (or greater than or equal to), or not equal to signs are used in place of the equal sign.

Value of gift card=$25

Cost of each song=$1.29

Let m be the number of songs purchased.

Then, the required inequality is as follows:

[tex]1.29m\leq 25\\m\leq \frac{25}{1.29}\\ m\leq 19.38[/tex]

The value of m rounded to the nearest whole number is 19.

Learn more about inequality here:

https://brainly.com/question/24372553

#SPJ4

A newspaper article about the results of a poll states: "In theory, the results of such a poll, in 99 cases out of 100 should differ by no more than 5 percentage points in either direction from what would have been obtained by interviewing all voters in the United States." Find the sample size suggested by this statement.
Group of answer choices

27

664

544

385

Answers

The sample size suggested by this statement is 664 option second is correct.

What is the margin of error(MOE)?

It is defined as an error that provides an estimate of the percentage of errors in real statistical data.

The formula for finding the MOE:

[tex]\rm MOE = Z\times \dfrac{s}{\sqrt{n}}[/tex]

Where   Z is the z-score at the confidence interval

            s is the standard deviation

            n is the number of samples.

At 99% confidence Z = 2.576

By using the MOE formula:

0.05 = 2.376×√(0.5(0.5)/n)

After calculating

n = 663.57 ≈ 664

Thus, the sample size suggested by this statement is 664 option second is correct.

Learn more about the Margin of error here:

brainly.com/question/13990500

#SPJ1

Triangle WXY has the interior and exterior angles shown below.

Triangle X W Y. Angle W is (2 x minus 13) degrees and angle Y is x degrees. The exterior angle to angle X is 122 degrees.

What is Measure of angle W?
45 degrees
58 degrees
68 degrees
77 degrees

Answers

The correct option is last one  77 degrees.

Therefore the measure angle W is 77 degrees.

Step-by-step explanation:

Given:

Exterior angle = 122°

∠W = (2x-13)°

∠Y= x°

To Find:

∠W = (2x-13)° = ?

Solution:

Exterior Angle Property:

An exterior angle of a triangle is equal to the sum of the opposite interior angles.

Angle W and Angle Y are the interior angles of Exterior angle 122°.

Substituting the values we get

Now Substitute 'x' in angle W we get

Therefore the measure angle W is 77 degrees.

Step-by-step explanation:

f(x) = ^3√4x
g(x) = 3x + 5
Find (f/g)(x). Include any restrictions on the domain.

Answers

The required function (f/g)(x) is 3√4x/2x+3 where x ≠ -3/2

How to Simplify Algebraic Functions?

Given the functions;

f(x) = ∛4x

g(x) = 2x + 3

We want to find the resulting function (f/g)(x);

(f/g)(x) = f(x)/g(x)

Thus, (f/g)(x) = ∛4x/(2x + 3)

Restriction to the function occurs at the point where the denominator is equal to zero.

Thus;

2x + 3 = 0

2x = -3

x = -3/2

Thus, the required function (f/g)(x) is 3√4x/2x+3 where x ≠ -3/2

Read more about Restriction Function at; https://brainly.com/question/23718282

#SPJ1

On a coordinate plane, triangle T V W has points (negative 4, 8), (0, 4), and (4, 4).

What are the coordinates of the endpoints of the segment T'V'?
T'(-3, 6) and V'(0, 3)
T'(-3, 6) and V'(0, 1)
T'(-1, 2) and V'(0, 3)
T'(-1, 2) and V'(0, 1)

Answers

The coordinates of the endpoints of line segments T'V' are; T'(-1, 2) and V'(0, 1).

What are the coordinates of the endpoints of the segment T'V'?

It follows from the task content that the transformation involved in the formation of the image from the pre-image is dilation by a scale factor of 1/4.

On this note, given that the coordinates of T and V from the task content are; (-4, 8) and (0,4), it follows that the coordinates of the endpoints as required are; T'(-1, 2) and V'(0, 1).

Read more on dilations;

https://brainly.com/question/3457976

#SPJ1

Use the properties of 30-60-90 and 45-45-90 triangles to solve for x in each of the problems below. Then decode the secret message by matching the answer with the corresponding letter/symbol from the exercises.

Answers

These problems are solved using the trigonometric function. Trigonometric functions provides the ratio of different sides of a right-angle triangle.

What are Trigonometric functions?

The trigonometric function refer to function that are periodic in nature and which lend insight to the relationship between angles and the sides of a triangle that is right angled.

The solutions to x in the respective problems is given as follows:

1st.) x = 5 /Sin(30°)

x = 10

!) sin(45°) = 4/x

x = 4/sin(45°)

x = 4√2

I) Cos(45°) = √3 / x

x = √3 / Cos(45°)

x = √6

E) Tan(60°)

= (3√3) / x

x = (3√3) / 3

W) It is to be noted that for right-triangle that is isosceles in nature, the angle made by the legs and the hypotenuse is always 45°.

x = 45°

N) x² + x² = (7√2)²

x = 7

V) Tan(60°) = 7 / x

x = 7√3/3

K) x² + x² = (9)²

x = 9/√2

Y) Sin(60°) = 7√3/x

x = 14

M) Sin(30°) = x/11

x = 11/2

T) Sin(45°) = x/√10

x = √5

A) x + 2x + 90° = 180°

x = 30°

O) Sin(45°) = √2 / x

x = 2

R) Tan(30°) = x / 4

x = 4/√3

= 4√3 / 3

S) Sin(60°) = x / (10/3)

x = (5√3) / 3

Learn more about Trigonometric functions at:
https://brainly.com/question/1143565
#SPJ1

A student submitted the following work that is not correct. Can you find the mistake?
Explain the mistake the student made.
Simplify (assuming all variables are positive): √32x³y²z7
Step 1: √8.4.x².xy².26. z
Step 2: 2xyz³ √8xz

Answers

Step-by-step explanation:

so you have the equation: [tex]\sqrt{32x^3y^2z^7}[/tex] which I'm assuming is what you meant to input. and the next step I'm assuming you wrote: [tex]\sqrt{8} * \sqrt{4} * \sqrt{x^2} * \sqrt{y^2} * \sqrt{x} * \sqrt{z^6} * \sqrt{z}[/tex], although I'm not 100% sure, I'm basing this assumption off of step 3. Anyways this will simplify to [tex]2xyz^3\sqrt{8xz}[/tex] which is correct kind of, but since they were asked to FULLY SIMPLIFY, then it's not completely done. This is because 8 has a factor of 4 which is a perfect square. This is because in step 1, the student rewrote 32 as sqrt(8) * sqrt(4) instead of writing it as sqrt(2) * sqrt(16) which is the greatest factor of 32 that is a perfect square. So they could've either done that in step 1, or they could've realized in step 2, that they can further simplify sqrt(8) and then have a step 3 showing that.

What is the difference between the sum of the first 2003 even counting numbers and the sum of the first 2003 odd counting numbers

Answers

The difference is 2003.

The formula for the sum of the first n even numbers is SE = n^{2} + n, (E standing for even).

The sum of the first n odd numbers is SO = n^{2}, (O standing for odd).

Knowing this, plug 2003 for n,

SE - SO= (2003^{2} + 2003) - (2003^{2}) = 2003

The difference is 2003.

A number that can be divided into two halves, i.e. into two equal parts is called an even number. Even numbers are exactly divisible by 2 which means the remainder will be 0.

Learn more about Even numbers here: https://brainly.com/question/251701

#SPJ4

Myra took a picture of the sky one afternoon when two jet airplanes appeared to draw a pair of parallel lines with their vapor trails. The vapor trails from two other jets flying from another direction crossed over the parallel trails. She printed her picture and labeled the angles and lines.
Parallel lines c and d are cut by transversals a and b. All angles are described clockwise, from uppercase left. The intersection of lines c and b form angles: 2, 4, 3, 1. The intersection of lines d and b form angles: 6, 8, 7, 5. The intersection of lines c and a form angles: 10, 12, 11, 9. The intersection of lines a and d form angles: 14, 16, 15, 13.

Assume lines c and d are parallel and Angle2 measures 98°. Which statements are true? Select three options.
mAngle3 = mAngle6 = 98°
mAngle3 = mAngle14 = 98°
mAngle4 = mAngle8 = 82°
mAngle4 = mAngle12 = 82°
mAngle5 = mAngle8 = 82°

Answers

Options 1, 3 and 5. The true statements based on the fact that c and d are parallel are:

m<3 = m<6 = 98 degrees m<4  = m<8 = 82degreesm<5 = m<8 = 82 degrees

How to solve for the angles

The sum of the angles m<4 + m<3 = 180

m<4 + 82 = 180

m<4 = 180 - 82= 98

From the question we have the first as alternate interior angles. The second is corresponding angles and the last option is vertical angles.

Read more on angles here:

https://brainly.com/question/26033691

#SPJ1

3. Donnell is doing market research for a major brand by surveying customers at a local mall.
The probability of a customer agreeing to take the survey is 22%. What is the probability that
one of the first five customers agrees to take the survey?
29%
54%
071%
74%

Answers

71% is the required probability here.

What is a probability simple definition?

A probability is a numerical representation of the likelihood or chance that a specific event will take place. Both proportions ranging from 0 to 1 and percentages ranging from 0% to 100% can be used to describe probabilities.

                                        The probability serves as a gauge for the possibility that an event will occur. and a The probability equation is stated by the following notation: P(E) = Number of Favorable Outcomes/Number of Total Outcomes.

The number of customers out of 5 who agree to take the survey here could be modelled as a binomial distribution given as

 x ` Bin (n = 5 , p = 0.22 )

Thus the probability here is computed as

P(X≥ 1) = 1 - P(X = 0) = 1 - (1 - 0.22)⁵

           = 0.71 %

Therefore 71% is the required probability here.

Learn more about probability

brainly.com/question/30034780

#SPJ1

Beth wanted to buy vegetables. She noticed that brand A of black beans was on sale for 4 cans for 87 cents. Brand B was 3 cans for 79 cents. Which was a better buy?

Answers

Answer:

Brand A

Step-by-step explanation:

If you divide both brands to find how much one can costs, brand B costs more per can.

79 ÷ 3 = 26.34

87 ÷ 4 = 21.75

That being said, if brand B were to sell 4 cans instead of 3 it would be more than brand A's 4 cans. Brand A is cheaper than brand B would be.

The way you answer the question is to find out the price of just one can so you divide each total cost by the number of cans.

$.87\4 =$.2175 brand A

$.79/3= $ .263 brand

Thus Brand A costs less per can and is the better buy.

How to find the size of an exterior angle of a regular polygon

Answers

Answer:

exterior angle of a polygon = 360° ÷ number of sides

Step-by-step explanation:

exterior angle of a polygon = 360° ÷ number of sides

two numbers that equal -8 when multiplying but thoses two numbers also have to equal to -7 when adding

Answers

Answer:

-8 and 1

Step-by-step explanation:

-8 x 1 is -8

-8+1 is -7

The function f(x) is given by the set of ordered pairs.

Answers

The equation that is true and one that matches the set of ordered pairs of functions is option C. f (0) = 6

What is the equation about?

Note that a function can be made to be a given set of ordered pairs such as the ordered pairs (2,4), (3,5), (4,6)  can have  input functions of ​​(as domain) 2,3,4 and output functions of ​​(as range) of 4,5,6.

So when you see the ordered pairs. {(1,0), (–10,2), (0,6), (3,17), (–2, –1)} one can see that the option that matches to the set of ordered pairs of functions above is f (0) = 6 value x = 0, f (x) = y = 6, ordered pairs (0.6)

See full question below

The function f (x) is given by the set of ordered pairs. {(1,0), (–10,2), (0,6), (3,17), (–2, –1)}:Which equation is true?

f(-10)=1

f(2)=-10

f(0)=6

f(1)=-10

The answer choices are :

f (-10) = 1 values ​​x = -10, f (x) = y = 1, ordered pairs (-10,1)

f (2) = - 10 values ​​x = 2, f (x) = y = -10, ordered pairs (2, -10)

f (0) = 6 value x = 0, f (x) = y = 6, ordered pairs (0.6)

f (1) = - 10 values ​​x = 1, f (x) = y = -10, ordered pairs (1, -10)

Learn more about ordered pairs from

https://brainly.com/question/6200217

3SPJ1

which of these ratios are equivalent ratio
1:4 2:8 3:6 7:28

Answers

Answer:

1:4

2:8

7:28

Step-by-step explanation:

1:42:8 , 1:47:28, 1:4

Mark is investing his money. he thinks that he should make $10 for every $100 he invests. how much does he expect to make on an investment of $500?

Answers

$50
Work: 500 / .10 = 50

Use the figure to decide the type of angle pair that describes ∠5 and ∠6.

Answers

Answer: Corresponding angles.

Reason:

These angles are both in the same corresponding direction. Both are in the same western corner, or point to the west. If the lines were parallel, then the corresponding angles would be congruent.

Suppose a and b are real numbers such that 17^a=16 and 17^b=4. What is 1/(2^a-b) ?

Answers

Since

[tex]17^a = 16 = 4^2 \implies 17^{a/2} = 4[/tex]

it follows that

[tex]17^b = 4 \implies 17^{a/2} = 17^b \implies \dfrac a2 = b \implies a = 2b[/tex]

Then

[tex]2^{a - b} = 2^{2b - b} = 2^b[/tex]

so that

[tex]\dfrac1{2^{a-b}} = 2^{-b}[/tex]

We also have

[tex]17^b = 4 \implies b = \log_{17}(4)[/tex]

so we can go on to say

[tex]\dfrac1{2^{a-b}} = \boxed{2^{-\log_{17}(4)}}[/tex]

A certain positive integer has exactly 20 positive divisors. What is the smallest number of primes that could divide the integer

Answers

As per my explanation to (b) above, the largest number of primes that could factor such a number is 4.

Note that  2,3,5 and 7   are the smallest primes, then use the reasoning from

(b) above. we are looking for four exponents, that, when 1 is added to each and all are multiplied together, would equal 20.

But no such integers  k, l, m, and n  exist such that (k + 1)(l + 1)(m + 1) (n + 1)  = 20 where k, l,m, and n ≥ 1 so this number, whatever it is, can't have 4 prime factors

Let's drop 7 out of the mix and suppose it has just 3 prime factors 2, 3, and 5 again we are looking for three exponents,  that, when 1 is added to each and all are multiplied together, would equal 20.  Put another way, we are looking for k, l and m ≥ 1   such that (k + 1)(l + 1)(m + 1) = 20

Note that the  only possibility here  is when we have 2 *2 *5  = 20 and the smallest possible product would be 2^(4 )* 3^(1) * 5^(1) =  2^4 * 3 * 5 = 240

Now......the only remaining possibility is  that this number is composed of the two smallest primes, 2 and 3,  and we are looking for  some k  and l ≥ 1 such that (k + 1)(l + 1) = 20 clearly, the only possibilities  are when k = 4 and l = 5, or vice-versa

So this number would factor as either 2^3 * 3^4   = 648  or 2^4 * 3^3 = 432 and both are > 240.

Learn more about positive integers at

https://brainly.com/question/1367050

#SPJ4

A traffic light runs repeatedly through the following cycle: green for 30 seconds, then yellow for 3 seconds, and then red for 30 seconds. Leah picks a random three-second time interval to watch the light. What is the probability that the color changes while she is watching

Answers

The probability that the colour changes, while she is watching, is 1/7

Concept: Event probability = number of favorite results /Total number of results

It is given that the traffic light runs repeatedly in the following time cycle: green for 30 seconds, then yellow at 3  seconds, and then red for 30 seconds.

This means that the traffic light passes through 63 seconds to complete one-time cycle.

It is also given that Leah chooses a random 3 second time interval to watch the light.

We should find the probability that the color changes while Leah is looking.

The green light will change after 30 seconds and after three seconds it will change to yellow light, that is at 33 second it is yellow and after another 30 seconds it will change to red light.

Now let time t=0

The light changes at three different times: t=30,t = 33 and t = 63

This means that the light will change if Leah watches at intervals [27,30]

, [30,33] or [60,63]

Since each interval is 3 seconds and there are 3 intervals, Leah will have a chance of 9 seconds.

That makes a total of 9 seconds out of 63

⇒Probability that Leah watches the color change=Number of seconds she watches/Total seconds

=9/63=17

∴ The probability that Leah observes the color change is 1/7

For more information about probability, visit https://brainly.com/question/24756209

#SPJ4

Find the condition of the expression:√3-2x-x^2

Answers

Answer:

-1 <= x <= 1

Step-by-step explanation:

√((1-2x+x^2)+2-2x^2)

= √((1-x)^2 + 2*(1-x)(1+x) )

điều kiện là 1 - x > =0 và 1+x >=0

giải 2 bất phương trình trên ta thu đc kết ququả

Identify the slope and y-intercept of the line whose equation is given. write the y-intercept as an ordered pair. y = negative 2 x + 5 a. slope = negative 2; y-intercept (0, 5) b. slope = 2; y-intercept (0, 5) c. slope = 5; y-intercept (0, 2) d. slope = negative 5; y-intercept (0, 2) please select the best answer from the choices provided a b c d

Answers

The line having the equation, y = -2x + 5, has a slope = -2, and y-intercept = (0, 5). Thus, option A is the best choice.

The slope of a line is the tangent of the angle that the line makes with the positive side of the x-axis.

The y-intercept of a line is the point at which the line intersects the y-axis.

The standard form of a line is y = mx + b, where m represents the slope of the line, and b represents its y-intercept.

In the question, we are asked to identify the slope and the y-intercept of the line whose equation is y = -2x + 5.

Comparing the given equation of the line, y = -2x + 5, with the standard equation of a line, y = mx + b, we can say that,

m = -2, and b = 5.

Representing b as an ordered pair, we get (0, 5), as b represents the y-intercept which is a point on the y-axis, where the x-coordinate is always equal to zero.

Thus, we get the slope = -2, and the y-intercept = (0, 5).

The option representing the answer is option A.

Learn more about the slope and the y-intercept of a line at

https://brainly.com/question/1479018

#SPJ4

Write an equation of the line that passes through a pair of points: on a coordinate plane, a line goes through (0, 2) and (2, 0). a. y = x + 3 c. y = -x + 2 b. y = x - 3 d. y = -x - 2 please select the best answer from the choices provided a b c d

Answers

Since the y-intercept b = 2, hence the equation of the line will be y = -x +2

Equation of a line

The formula for calculating the slope of a line is expressed as:

Slope = y2-y1/x2-x1

Given the coordinate points  (0, 2) and (2, 0)

Slope = 0-2/2-0
Slope = -1

Since the y-intercept b = 2, hence the equation of the line will be y = -x +2

Learn. more on equation of a line here: https://brainly.com/question/18831322

#SPJ1

Answer:

c

Step-by-step explanation:

What is 12.1975 rounded to the nearest thousandth?

Answers

Answer:

12.198

hope it helps

12.198

To round over 7 into 8, the ten-thousandth place needs to be 5 or over, and as we can see the ten-thousandth place is 5.

The length of the hypotenuse of a right triangle is 7 feet, and the length of one leg is 4 feet. what is the length of the other leg to the nearest tenth of a foot

Answers

The length of the other leg to the nearest tenth of a foot is 5.7 feet.

What is Pythagorean theorem?

A theorem in geometry: the square of the length of the hypotenuse of a right triangle equals the sum of the squares of the lengths of the other two sides.The Pythagoras theorem equation is expressed as, [tex]c^{2} = a^{2} + b^{2}[/tex], where 'c' = hypotenuse of the right triangle and 'a' and 'b' are the other two legs.

We can use the Pythagorean Theorem to find the length of the other leg

[tex]a^{2} + b^{2} = c^{2}[/tex]

"a" and "c" represent the two legs of the triangle and "b" represents the perpendicular.

So,

      [tex]b^{2} = c^{2} - a^{2}[/tex]

        =  7² - 4²

  b = [tex]\sqrt{49 - 16}[/tex]

  [tex]b = \sqrt{33}[/tex] ⇒ 5.7 feet

Therefore, the length of the other leg to the nearest tenth of a foot is 5.7 feet.

Learn more about Pythagorean Theorem brainly.com/question/14605061

#SPJ4

StartLayout 1st row StartRoot StartFraction 250 c cubed Over 9 d Superscript 6 Baseline EndFraction EndRoot space c greater-than-or-equal-to 0 and d greater-than-or-equal-to 0. 2nd row = StartRoot StartFraction 25 times 10 c squared times c Over 9 d Superscript 6 Baseline EndFraction EndRoot. 3rd Row = StartFraction StartRoot 25 EndRoot times StartRoot c squared EndRoot times StartRoot 10 c EndRoot Over StartRoot 9 d Superscript 6 Baseline EndRoot EndFraction EndLayout

Based on the work shown on the left, what is the simplest form of StartRoot StartFraction 250 c cubed Over 9 d Superscript 6 Baseline EndFraction EndRoot ?

Answers

The simplest form of the given algebraic expression √(250c³/9d⁶) is; 5c * (√10c)]/3d³

How to simplify algebra problems?

We are given the algebra problems as;

√(250c³/9d⁶)

Now 250c³ can be broken down into 250 = 25c² × 10c

Thus, we now have;

√(250c³/9d⁶) = [√(25c²) * (√10c)]/√9d⁶

When we breakdown the above expression, we have;

5c * (√10c)]/3d³

Read more about Algebra Problems at; https://brainly.com/question/723406

#SPJ1

Answer:

the answer is a

Step-by-step explanation:

i got it right

Sean used cross multiplication to correctly solve a rational equation. he found one valid solution and one extraneous solution. if 1 is the extraneous solution, which equation could he have solved?

Answers

The equation which he have solved is [tex]10/x^{2} -1=15/3x-3[/tex] as in this 1 is the extraneous solution.

Given four equations in which first equation is [tex]10/x^{2} -1=15/3x-3[/tex],second is (x+2)/(x+3)=6x/8, third is (4x-4)/(x+6)=x-1/10, fourth is 4/x-1=x+2/10.

We have to find such a equation which have a valid solution and one extraneous solution.

We have to solve these equation:

[tex]10/x^{2} -1=15/3x-3[/tex]

The above exprssion is not defined for x=1 so 1 is an extraneous solution. Simplifying the above equation by doing cross multiplication.

10(3x-3)=15([tex]x^{2} -1[/tex])

30(x-1)=15(x-1)(x+1)   [We know that [tex]A^{2} -B^{2} =(A+B)(A-B)[/tex]

2=x+1

x=1

In the given equation 1 makes the denominator zero and 1 is the solution of the equation derived from cross multiplication.

Hence the correct equation which Sean solves is option A which is [tex]10/x^{2} -1=15/3x-3[/tex].

Learn more about equation at https://brainly.com/question/2972832

#SPJ4

The given question is incomplete as it should include the given figure.

Answer:see photo #15

Step-by-step explanation:

The following three shapes are based only on squares, semicircles, and quarter circles.
Find the perimeter and the area of each shaded part.

Answers

The perimeter of the shaded part is: 25.1 cm.

The area of the shaded part is: 36.6 cm².

How to Find the Perimeter and Area of the Shaded Part?

Area = Area of square - 2(area of square - area of quarter circle) = s² - 2(s² - 1/4πr²).

Parameters given are:

s = 8 cmr = 8 cm

Plug in the values into the equation

Area = 8² - 2(8² - 1/4π(8²)).

Area = 64 - 2(64 - 50.3)

Area = 36.6 cm²

Perimeter = 2(perimeter of quarter circle) = 2(1/4(2πr)) = 2(1/4(2π8)) = 25.1 cm

Learn more about perimeter and area on:

https://brainly.com/question/443376

#SPJ1

Terrell brought 42 muffins to school for his birthday He gave one each to the 17students in his class and to the 19 student in the other third grade class

Answers

Answer:

The expression is 42-17-19 OR 42-(17+19).

Step-by-step explanation:

First, we can indicate that we can use subtraction because is giving them away for his birthday.

"He gave one each to the 17 students in his class and to the 19 students in the other third-grade class".

That represents taking away 17 and 19 means that it is subtracting them.

Hope this helps. (:

E represents a number. For example, if E is 5, EE equals 55 and 4E equals 45. If 1E x E = 9E, what number should be in the place of E?

Answers

Answer:

E represents a number, for this question we need to keep in mind that the product of 1E and E must end in E.

E must be 6.

E = 6

Step-by-step explanation:

16 * 6 = 96

Other Questions
Longhorn Corporation provides low-cost food delivery services to senior citizens. At the end of the year on December 31, 2021, the company reports the following amounts:Cash $ 1,300 Service revenue $ 65,700 Equipment 27,000 Cost of goods sold (food expense) 53,000 Accounts payable 4,000 Buildings 36,000 Delivery expense 2,200 Supplies 3,000 Salaries expense 5,100 Salaries payable 900 In addition, the company had common stock of $36,000 at the beginning of the year and issued an additional $3,600 during the year. The company also had retained earnings of $17,400 at the beginning of the year.Prepare the statement of stockholders equity for Longhorn Corporation. Name the mineral in meat that is needed to make haemoglobin. What tool does a programmer use to produce Python source code? A 2-column table with 5 rows. The first column is labeled x with entries negative 2, negative 1, 0, positive 1, positive 2. The second column is labeled f(x) with entries negative 13, negative 10, negative 7, negative 4, negative 1. What is the rate of change of the linear function represented by the table? StartFraction one Over three EndFraction StartFraction one Over two EndFraction 2 3 If one line passes through the points open parentheses negative 3 comma space minus 5 close parentheses space and open parentheses 1 comma space 3 close parentheses space and another line passes through the points open parentheses 0 comma space 1 close parentheses space and open parentheses 2 comma space 0 close parentheses determine if the two lines are parallel, perpendicular, or neither. Washing your hands should take at least a minute? 65 people took a clarinet exam. Before the exam, 23 people predicted that they would pass, and the rest predicted they would fail. Of the people who predicted they would pass, 18 actually did. In total, 31 people passed the clarinet exam. What fraction of those who thought that they would fail actually did fail? In a resting state, sodium (Na ) is at a higher concentration outside the cell and potassium (K ) is more concentrated inside the cell. During an action potential, the sodium levels ________ inside the cell. The first eight ionization energies of a third row element are 999.6, 2252, 3357, 4556, 7004, 8496, 27107, and 31719 kJ/mol. Identify the element. Mr. Suksod drives from his house to the office at a rate of speed of 82 km/h in 15 minutes. Mr.Taylor, whose house is as far from the office as Mr. Suksod's, drives for 10 minutes to reach thesame office. What is the difference in the rate of speed that they drive? what are the types of resources required for the development describe How to care for the teeth atmosheric concentraion of green house gases such as carbon dioxide are higher than at any time in earth history. T/F There are 8 apples in a small fruit basket and 14 apples in a large fruit basket. If Sally buys a total of 5 fruit baskets that have a combined total of 58 apples, write an equation that represents the total number of apples that Sally has. please can you hurryWhat is the truth value for the following conditional statement?p: trueq: falsep qANSWERST F TF F TT F FF T T Tavon has a gift card for 80$ that loses $3.50 for each 30-day period it is not used. He has another gift card for $70 that loses $3 for each 30-day period it is not used. Kevin built a wooden flower box for his garden. The area of the base of the box is 264 square centimeters. If the rectangular base has the dimensions shown below, whatis the value of x? Given Principal = $11,500, Interest Rate = 10%Time = 240 days (use ordinary interest)Partial payments:On 100th day, $6,000On 180th day, $3,500a. Use the U.S. Rule to solve for total interest cost. (Use 360 days a year. Do not round intermediate calculations. Round your answer to the nearest cent.)b. Use the U.S. Rule to solve for balances. (Use 360 days a year. Do not round intermediate calculations. Round your answers to the nearest cent.)c. Use the U.S. Rule to solve for final payment. (Use 360 days a year. Do not round intermediate calculations. Round your answer to the nearest cent.) Complete these sentences with personal pronouns. 2. This camera takes excellent pictures. I like _____ very much. 3. The birthday boy gave sweets to his friends and ______ thanked ______.4. Mother said, "Varun, must help______ in the kitchen. 5. ______completed his work and then helped others. 6. We have a good teacher. _____ advised ______ to work harder. ______always listen to ______.7. My father told me, "I want ______to take these papers and put _______in the folder . give them to _______ when _______ ask for them." Consider the debate about whether viruses are alive or not. Both Viewpoint #1 and Viewpoint #2 have points of evidence in common. Evidence includes ALL BUT _____.